36 0 401KB
CH 1_2 Exercices SNP 10/01/2022
Exercice 1 Dans une étude américaine sur l’addiction aux drogues dans une ville A, on a constaté que le QI médian chez les drogués âgés d’au moins 16 ans était de 107. Pour tester que ce résultat est valable dans une ville B, voici les QI de 12 drogués d’au moins 16 ans vivants dans B: 108; 111; 99; 127; 109; 100; 90; 94; 135; 104; 119; 117 Au seuil de signification α = 0.05, que peut-on conclure?
Solution (Test des Signes) Notons md le QI médian. On voudrait tester l’hypothèse H0 : md = 107 contre l’elternative H1 : md = 6 107. On utilise alors le test des signes. scrQI=c(108,111,99,127,109,100,90,94,135,104,119,117) sgn_scrQI=sign(scrQI-107) Spls=sum(sgn_scrQI>0) On rejette H0 pour les trop grandes ou trop petites valeurs de S + .On constate que S + = 7, donc la p-value=2*P{S + > 7}. pvl=2*pbinom(Spls,length(scrQI),0.5,lower.tail = FALSE) pvl #([1] 0.3876953) Avec une p-value=0.3877, on ne rejette pas H0 . On conclut donc qu’il n’y a pas suffisamment d’évidence que md 6= 107.
Exercice 2 Les données ci-après sont les densités (en gr/cm2 ) de 15 feuilles choisies au hasard dans une ramette de papier. 81.6; 83.6; 81.5; 82.2; 82.2; 84.3; 82.4; 78.8; 78.2; 79.5; 80.5; 83.6; 73.8; 77.2; 77.6. On suppose que la densité d’une feuille de papier suit une loi de probabilité symétrique. Au seuil de signification α = 0.05, peut-on conclure qu’au moins 50% des feuilles de la ramette ont une densité d’au moins 80.0 gr/cm2 ?
1
Solution (Test des rangs signés de Wilcoxxon) Notons md la densité médiane. On voudrait tester l’hypothèse H0 : md ≥ 80 contre l’elternative H1 : md < 80. On peut utiliser le test des signes et rejeter H0 pour les petites valeurs de S + . Il est également possible d’utiliser le test des rangs signés de Wilcoxon. Ce dernier est bien entendu préférable au premier. On rejette H0 pour les petites valeurs de W +. dnst=c(81.6,83.6,81.5,82.2,82.2,84.3,82.4,78.8,78.2,79.5, 80.5,83.6,73.8,77.2,77.6) dnst-80 rank(abs(dnst-80)) rnkdnst=ifelse(dnst-80>0,rank(abs(dnst-80)),0) Wpls=sum(rnkdnst) Wpls
Outputs Les différences: 1.6 3.6 1.5 2.2 2.2 4.3 2.4 -1.2 -1.8 -0.5 0.5 3.6 -6.2 -2.8 -2.4 Les rangs des valeurs absolues des différences: 5.0 12.5 4.0 7.5 7.5 14.0 9.5 3.0 6.0 1.5 1.5 12.5 15.0 11.0 9.5 Les rangs des valeurs absolues des différences positives: 5.0 12.5 4.0 7.5 7.5 14.0 9.5 1.5 12.5 Wpls= 74 D'après la table: P{Wols>= 74}=0.227. D'où la p-value=1-P{Wols>= 74}=0.773. Il n’y a donc pas suffisamment d’évidence pour rejeter H0 . On peut donc conclure qu’au moins 50% des feuilles de la ramette ont une densité supérieure ou égale à 80.0 gr/cm2 .
Solution avec R dnst=c(81.6,83.6,81.5,82.2,82.2,84.3,82.4,78.8,78.2,79.5, 80.5,83.6,73.8,77.2,77.6) wilcox.test(dnst,mu=80,alternative = "less",exact = TRUE) Wilcoxon signed rank test with continuity correction data: dnst V = 74, p-value = 0.7951 alternative hypothesis: true location is less than 80
2
Exercice 3 Le tableau suivant contient les niveaux d’une composante chimique dans le sang de dix malades avant (Av) et après (Ap) l’injection d’une dose d’un certain traitement. Av Ap
0.61 0.56
0.59 0.46
0.68 0.59
0.67 0.53
0.70 0.79
0.56 0.50
0.64 0.54
0.59 0.52
0.64 0.72
0.64 0.60
Au seuil de signification α = 0.05, peut-on conclure que le traitement réduit le niveau de la composante chimique dans le sang des malades?
Solution Il s’agir de deux échantillons appariés. On considère l’échantillon des différences D=Avant-Après, et on teste l’hypothèse H0 : mD = 0 contre l’hypothèse H1 : mD > 0, où mD est la médiane des différences. On suppose que la loi des différence est symétrique1 par rapport à mD et on applique le test des rangs signés de Wilcoxon W + . On rejette H0 pour les grandes valeurs de W + . avnt=c(0.61, 0.59, 0.68, 0.67, 0.70, 0.56, 0.64, 0.59, 0.64, 0.64) aprs=c(0.56, 0.46, 0.59, 0.53, 0.79, 0.50, 0.54, 0.52, 0.72, 0.60) dffr=avnt-aprs Wpls=sum(rank(abs(dffr))*(dffr>0)) Wpls On constate que W + = 43.5, d’près la table, on a p-value=P{W pls > 43.5} = 0.053. On conclut donc, au seuil α = 0.05, qu’il n’y a pas suffisamment d’évidence que que le traitement réduit le niveau de la composante chimique dans le sang des malades.
Solution avec R avnt=c(0.61, 0.59, 0.68, 0.67, 0.70, 0.56, 0.64, 0.59, 0.64, 0.64) aprs=c(0.56, 0.46, 0.59, 0.53, 0.79, 0.50, 0.54, 0.52, 0.72, 0.60) wilcox.test(avnt,aprs,paired = TRUE, alternative = "g",exact = TRUE) Wilcoxon signed rank test with continuity correction data: avnt and aprs V = 43.5, p-value = 0.05695 alternative hypothesis: true location shift is greater than 0 avnt=c(0.61, 0.59, 0.68, 0.67, 0.70, 0.56, 0.64, 0.59, 0.64, 0.64) aprs=c(0.56, 0.46, 0.59, 0.53, 0.79, 0.50, 0.54, 0.52, 0.72, 0.60) dffr=avnt-aprs sign(dffr) Spls=sum(sign(dffr)>0) Spls pvl=pbinom(Spls,length(avnt),0.5,lower.tail = FALSE) pvl 2.888 rank(abs(dffr)) Wpls=sum(rank(abs(dffr))*(dffr>0)) Wpls 1 Cette
hypothèse est faite dans le but de pouvoir illustrer les calculs pour le test des rangs signés de Wilcoxon
3
Exercice 4 Selon la théorie des tests psychologiques, les scores d’un certain test du QI sont normalement distribués. Ce test a été subi par 18 personnes à peu près de même âge. Voici leurs scores: 114; 81; 87; 114; 113; 87; 111; 89; 93; 108; 99; 93; 100; 95; 93; 95; 106; 108. Tester la normalité de ces scores si, a) la moyenne et la variance sont inconnues, b) la moyenne est µ = 100 et la variance est σ 2 = 100.
Solution dnn=c(114,81,86.999,114,113,87,111,89,92.999,108,99,93,100,95,93,95,106,108) (sort(dnn)-100)/10 fxi=c(pnorm((sort(dnn)-100)/10,mean = 0,sd = 1)) fxi suit=c(1:18)/18 suit abs(suit-fxi) D=max(abs(suit-fxi)) ks.test(dnn,"pnorm",100,10) shapiro.test(dnn)
Exercice 5 Dans un camp d’entrainement de basketball, les participants sont classés en trois catégories A, B ou C selon leur performances. Historiquement, les pourcentages des participants classés dans ces catégories sont respectivement 80%, 12% et 8%. Cette année dans un échantillon de 200 participants, 136 sont classés A, 38 sont classés B et 26 sont classés C. Peut-on conclure que les pourcentages n’ont pas changé? (utiliser α = 0.05).
Solution Soit X la variable qui désigne la classe d’un participant choisi au hasard et π = (πA , πB , πC ) sa loi de probabilité. On cherche à tester l’hypothèse H0 : π = (0.80, 0.12, 0.08) contre l’hypothèse H1 : π 6= (0.80, 0.12, 0.08). On utilisera le test d’ajustement de khi-deux.
4
ctgr=c("A"=136,"B"=38,"C"=26) Hprb=c(0.80,0.12,0.08) fresp=200*Hprb ki2=sum((ctgr-fresp)ˆ2/fresp) ki2 pchisq(ki2,df= 2,lower.tail = FALSE)
Outputs Résultats du Calcul à la main: khi2=18.01667 p-value= 0.0001223857 Avec une si petite p-value, il y a suffisamment d’évidence pour rejeter H0 et conclure que les pourecentage ont changé.
Solution avec R ctgr=c("A"=136,"B"=38,"C"=26) Hprb=c(0.80,0.12,0.08) chisq.test(ctgr,p=Hprb) Chi-squared test for given probabilities data: ctgr X-squared = 18.017, df = 2, p-value = 0.0001224
Exercice 6 Un contrôleur de qualité dans une chaîne de production a prélevé au hasard, dans la production du jour, 50 échantillons de taille 13 chacun. Le tableau ci-dessous montre la distribution du nombre d’item défectueux: Nombre d’items défectueux Effectif
0 9
1 26
2 9
3 4
4 1
5 1
6 ou plus 0
(a) Tester l’hypothèse H0 : “Le nombre d’item défectueux” suit une loi de Poisson. (b) Tester l’hypothèse H0 : “Le nombre d’item défectueux” suit une loi Binomiale. (c) Comparer les résultats de (a) et (b).
Solution (a) ˆ = 1.3. Avec cette valeur de λ on calcule les On estime λ par la moyenne de l’échantillon; on a alors λ probabilités des valeurs observées:
5
x P(x) 50 ∗ P(x)
0 0.27 13.50
1 0.35 17.50
2 0.23 11.50
3 0.10 5.00
4 0.03 1.5
5 0.01 0.5
6 ou plus 0.00 0.00
Après regroupement, on a: x P(x) FE:=Fréquence espérée=50 ∗ P(x) FO:=Fréquence Observée (FE-FO)2 /FE χ2 =
0 0.27 13.50 9 1.50 6.32
1 0.35 17.50 26 4.13
2 0.23 11.50 9 0.54
3 ou plus 0.10 7.00 6 0.20
La p-value est égale à 0.097. On ne rejette donc pas l’hypothèse H0 au seuil α = 0.05.
Solution (b) Pour tester H0 dans ce cas, on a besoin d’estimer p la probabilité de succès (obtenir une pièce défectueuse). Il s’agit de 50x13=650 vérifications, donc d’un échantillon de taille 650 où on a constaté un 65 total de 65 pièces défectueuses (9x0+26x1+9x2+4x3+1x4+1x5=65). Ainsi pˆ = = 0.1. 650 Selon l’hypothèse H0 , les 50 observations sont issues d’une loi binômiale avec n = 13 et p = 0.1.
Solution (b) Avec cette valeur de p on calcule les probabilités des valeurs observées: x P(x) 50 ∗ P(x)
0 0.25 12.50
1 0.37 18.50
2 0.24 12.00
3 0.10 5.00
4 0.03 1.50
5 0.01 0.50
6 ou plus 0.00 0.00
Après regroupement, on a: x P(x) FE:=Fréquence espérée=50 ∗ P(x) FO:=Fréquence Observée (FE-FO)2 /FE χ2 =
0 0.25 12.50 9 0.98 4.91
1 0.37 18.50 26 3.04
2 0.24 12.00 9 0.75
3 ou plus 0.14 7 6 0.14
La p-value est égale à 0.18. On ne rejette donc pas l’hypothèse H0 au seuil α = 0.05.
Solution (c) Dans les deux cas on ne rejette pas H0 ce qui semble un peu contradictoire. Cette contradiction n’est en fait qu’apparente puisqu’on sait que lois binomiale Bin(n, p) peut être approximée par une loi de Poisson de paramètre λ = np.
6
Exercice 7 Dans une étude de robustesse, les composantes d’avion sont soumises à un test de vibration jusqu’à ce qu’elles se brisent. Les données suivantes sont les temps de résistance aux vibrations (en mn) observés sur un échantillon de 15 composantes d’avion. 1.6; 10.3; 3.5; 13.5; 18.4; 7.7; 24.3; 10.7; 8.4; 4.9; 7.9; 12.0; 16.2; 6.8; 14.7 Au seuil de signification α = 0.05, peut-on conclure que cet échantillon provient d’une loi exponentielle de moyenne λ = 10?
Solution Nous allons utliser le test d’ajustement de Kolmogorov-Smirnov. 15
D = sup |F (x) − Fn (x)| = max
sup
k=0 xk ≤x 0.10. Donc on ne rejette pas H0 au seuil α = 0.05.
Exercice 8 Un ophtalmologue teste une nouvelle technique chirurgicale pour traiter le Glaucome (une pression intraoculaire élevée (IOP)). Il opère un œil du patient et laisse l’autre pour le contrôle. L’œil choisit pour l’opération est celui dont la pression est la plus élevée. Le tableau suivant contient les différences des mesures de l’IOP pour 9 patients avant et six mois après l’opération: Traité Ctrl. Diff.
0.45 0.38 +
1.95 0.90 +
1.20 0.70 +
0.65 -0.40 +
0.98 0.47 +
-1.98 -1.30 -
1.80 1.34 +
-0.76 0.13 -
0.56 -0.40 +
Au seuil de signification α = 0.05, peut-on conclure que la technique utilisée par l’ophtalmologue est efficace?
Solution Il s’agit ici de deux échantillons appariés. Notons Di la différence entre la valeur observée sur l’oeil traité et celle de l’oeil de contrôle. On teste alors H0 : la technique n’est pas efficace versus H1 : la technique est efficace. Notons S + le nombre de signes “+’ ’. On doit rejeter H0 pour S + assez grand. Sous H0 , S + suit une loi Binomiale(9, 0.5), donc la p-value=P{Binomiale(9, 0.5) ≥ 7} = 0.0898. Au seuil de signification α = 0.05, il n’y a pas suffisamment d’évidence que la technique est efficace. 7
Exercice 9 Le secteur touristique est une composante importante dans le produit intérieur d’un pays. Il permet les échanges avec l’extérieur et aide à réduire la balance des payement. L’agence “Travel Market Yearbook’ ’ publie chaque année d’importantes données sur le tourisme. Le tableau ci-dessous contient les données annuelles sur le tourisme au États Unis entre 1970 et 1982. Au niveau de signification α = 0.01, peut-on détecter l’existence d’une tendance? Annnée Touristes (en Millions) Annnée Touristes (en Millions)
1970 12 362 1977 18 610
1971 12 739 1978 19 842
1972 13 057 1979 20 310
1973 13 955 1980 22 500
1974 14 123 1981 23 080
1975 15 698 1982 21 916
1976 17 523
Solution Pour étudier l’existence d’une éventuelle tendance, on note xt le nombre de touristes pour l’année t et on calcule les quantité xt+1 − xt pour t = 1970, · · · , 1081. On etudie le signe de ces différences pour se prononcer sur la tendance. On a alors: ++++++-++ ++On teste alors H0 : il n’y a aucune tendance versus H1 : il y a une tendance de croissance. Notons S + le nombre de signes “+’ ’. On doit rejeter H0 pour S + assez grand. Sous H0 , S + suit une loi Binomiale(12, 0.5), donc la p-value=P{Binomiale(12, 0.5) ≥ 10} = 0.0193. Au seuil de signification α = 0.05, on rejette H0 pour conclure qu’il y a suffisamment d’évidence de l’existence d’une tendance de croissance.
Exercice 10 Les données ci-après sont le nombre de fois (par an) un échantillon de personnes sont tombées malades. Aucune activité physique 12, 15, 8, 11, 9, 17
Une activité physique régulière 9, 5, 10, 3, 4, 2
Au seuil de signification α = 0, 05, y a t-il suffisamment d’évidence que la pratique régulière d’une activité sportive augmente l’immunité du corps contre les maladies ?
Solution On cherche à tester l’hypothèse H0 :l’activité sportive est sans effet contre H1 : l’activité sportive réduit le risque de maladie. On va utiliser le test des rangs de Wilcoxon. Si on note W la somme des rangs de l’échantillon des non sportifs, on doit rejeter H0 pour les grandes valeurs de W . On obtient W = 53.5, ainsi, d’après la table, la p-value=P{W > 53.5} = 0.008. Au seuil de signification α = 0.05, on rejette H0 pour conclure qu’il y a suffisamment d’évidence que l’activité sportive réduit le risque de maladie.
Solution avec R sns.actv=c(12, 15, 8, 11, 9,17) avc.actv=c( 9,5, 10, 3, 4, 2) V=wilcox.test(sns.actv,avc.actv,alternative ="greater" ) V Wilcoxon rank sum test with continuity correction data: sns.actv and avc.actv W = 32.5, p-value = 0.01236 alternative hypothesis: true location shift is greater than 0 NB: Le code R calcule la statistique U de Mann-Whitney.
8